PT57.S3.Q6 - Economist: As should be obvious

lsatyayylsatyayy Alum Member
edited May 2018 in Logical Reasoning 178 karma

Hi everyone. Has anyone worked this question? If so, could you explain to me why D is incorrect? It's supposed to be an easy question being in the earlier portion of the section, but it trapped me. I get it that A is correct, but I still don't see why D (Most workers are earning more than the current minimum wages.) is wrong. Wouldn't it be true that if D is true, raising minimum wage wouldn't hurt businesses?

Just 5 days before the test, everyone. Good luck to us!

https://7sage.com/lsat_explanations/lsat-57-section-3-question-06/

Comments

  • akistotleakistotle Member 🍌🍌
    9377 karma

    PT57.S3.Q6. https://7sage.com/lsat_explanations/lsat-57-section-3-question-06/

    The title says it's "#3," so could you maybe edit the title?

    (D) is incorrect because it is irrelevant to the argument. Even if most workers are earning more than the current minimum wage, raising the minimum wage significantly would hurt the businesses, and it could still cause an increase in unemployment.

    Let's suppose that there are 100 workers in the entire world. 49 workers receive $10/hour, which is the minimum wage, 51 workers receive above $10/hour. Now the minimum wage is $20/hour. If you can afford to pay only $10/hour, you have to lay people off regardless of how much 51 workers receive.

  • lsatyayylsatyayy Alum Member
    178 karma

    Hi. I thought I left a comment here. But I guess I didn't. Thank you!! Thank you akistotle!!! :)

Sign In or Register to comment.